Quantcast
  • Register
PhysicsOverflow is a next-generation academic platform for physicists and astronomers, including a community peer review system and a postgraduate-level discussion forum analogous to MathOverflow.

Welcome to PhysicsOverflow! PhysicsOverflow is an open platform for community peer review and graduate-level Physics discussion.

Please help promote PhysicsOverflow ads elsewhere if you like it.

News

PO is now at the Physics Department of Bielefeld University!

New printer friendly PO pages!

Migration to Bielefeld University was successful!

Please vote for this year's PhysicsOverflow ads!

Please do help out in categorising submissions. Submit a paper to PhysicsOverflow!

... see more

Tools for paper authors

Submit paper
Claim Paper Authorship

Tools for SE users

Search User
Reclaim SE Account
Request Account Merger
Nativise imported posts
Claim post (deleted users)
Import SE post

Users whose questions have been imported from Physics Stack Exchange, Theoretical Physics Stack Exchange, or any other Stack Exchange site are kindly requested to reclaim their account and not to register as a new user.

Public \(\beta\) tools

Report a bug with a feature
Request a new functionality
404 page design
Send feedback

Attributions

(propose a free ad)

Site Statistics

205 submissions , 163 unreviewed
5,047 questions , 2,200 unanswered
5,345 answers , 22,709 comments
1,470 users with positive rep
816 active unimported users
More ...

  Amplitudes in renormalized perturbation theory

+ 4 like - 0 dislike
1364 views

This question arose while reading Peskin and Schroeder, specifically, it arose in regards to the sum of diagrams above their Eq. (10.20) on pg. 326.

The context is $\phi ^4$ theory and they are using a vertex renormalization condition to compute the counterterm $\delta _\lambda$ corresponding to the coupling constant $\lambda$. To do this, they calculate the $4$-point amplitude up to one-loop order in perturbation theory. In the process of doing this, however, they do not seem to include any one-loop diagrams involving the counterterms. Indeed, they only seem to include counterterm diagrams up to tree level.

Why is this? It seems, at least naively, that if one is doing a one-loop computation, one should compute all one-loop diagrams, and not discriminate between those Feynman rules in the 'original' theory and those that only arise during renormalization.

This post imported from StackExchange Physics at 2014-08-23 05:00 (UCT), posted by SE-user Jonathan Gleason
asked Nov 16, 2013 in Theoretical Physics by Jonathan Gleason (265 points) [ no revision ]
retagged Aug 23, 2014

1 Answer

+ 1 like - 0 dislike

The bare four-point vertex is of order $\sim \lambda$, and the counterterm four-point vertex is of order $\sim \lambda^2$ (although multiplied by infinity). The perturbation is done by considering the terms with the same order in $\lambda$.

So, you need to consider a one-loop diagram with two bare vertices and a tree diagram with one counterterm at the same time.

This post imported from StackExchange Physics at 2014-08-23 05:00 (UCT), posted by SE-user Yuji
answered Nov 16, 2013 by Yuji (1,395 points) [ no revision ]
This was more or less what I was expecting, but when I went to check this myself, I didn't see how $\delta _\lambda$ was of order $\lambda ^2$. Indeed, according to Peskin and Schroeder's Eq. (10.17) on pg. 324, $\delta _\lambda =\lambda _0Z^2-\lambda$, which is just of order $\lambda$ . . . or am I missing something?

This post imported from StackExchange Physics at 2014-08-23 05:00 (UCT), posted by SE-user Jonathan Gleason
@JonathanGleason The conterterm is of order $\lambda$ and the couplings to make the amplitude one-loop will introduce addtional $\lambda$s.

This post imported from StackExchange Physics at 2014-08-23 05:00 (UCT), posted by SE-user Neuneck
@Neuneck I don't understand. The Feynman rule corresponding to the $4$-point counterterm vertex is just $-\mathrm{i}\, \delta _\lambda$, not $-\mathrm{i}\, \delta _\lambda \lambda$ or anything like this, so this entire counterterm diagram should just be of order $\lambda$. What am I missing?

This post imported from StackExchange Physics at 2014-08-23 05:00 (UCT), posted by SE-user Jonathan Gleason
@JonathanGleason I was looking at your $\delta_\lambda$ from your previous answer, which seems to be proportional to $\lambda$.

This post imported from StackExchange Physics at 2014-08-23 05:00 (UCT), posted by SE-user Neuneck
@Neuneck Yes, but I thought the point is that the total result should be proportional to $\lambda ^2$, not just $\lambda$.

This post imported from StackExchange Physics at 2014-08-23 05:00 (UCT), posted by SE-user Jonathan Gleason
@JonathanGleason I see your point.

This post imported from StackExchange Physics at 2014-08-23 05:00 (UCT), posted by SE-user Neuneck

Your answer

Please use answers only to (at least partly) answer questions. To comment, discuss, or ask for clarification, leave a comment instead.
To mask links under text, please type your text, highlight it, and click the "link" button. You can then enter your link URL.
Please consult the FAQ for as to how to format your post.
This is the answer box; if you want to write a comment instead, please use the 'add comment' button.
Live preview (may slow down editor)   Preview
Your name to display (optional):
Privacy: Your email address will only be used for sending these notifications.
Anti-spam verification:
If you are a human please identify the position of the character covered by the symbol $\varnothing$ in the following word:
p$\varnothing$ysicsOverflow
Then drag the red bullet below over the corresponding character of our banner. When you drop it there, the bullet changes to green (on slow internet connections after a few seconds).
Please complete the anti-spam verification




user contributions licensed under cc by-sa 3.0 with attribution required

Your rights
...